Difference between revisions of "2015 AMC 8 Problems/Problem 5"

(One intermediate revision by the same user not shown)
Line 1: Line 1:
 
==Problem==
 
==Problem==
  
Billy's basketball team scored the following points over the course of the first 11 games of the season. If his team scores 40 in the 12th game, which of the following statistics will show an increase?
+
Billy's basketball team scored the following points over the course of the first <math>11</math> games of the season. If his team scores <math>40</math> in the <math>12^{th}</math> game, which of the following statistics will show an increase?
  
 
<cmath>42, 47, 53, 53, 58, 58, 58, 61, 64, 65, 73</cmath>
 
<cmath>42, 47, 53, 53, 58, 58, 58, 61, 64, 65, 73</cmath>

Revision as of 16:01, 16 January 2021

Problem

Billy's basketball team scored the following points over the course of the first $11$ games of the season. If his team scores $40$ in the $12^{th}$ game, which of the following statistics will show an increase?

\[42, 47, 53, 53, 58, 58, 58, 61, 64, 65, 73\]

$\textbf{(A) } \text{range} \qquad \textbf{(B) } \text{median} \qquad \textbf{(C) } \text{mean} \qquad \textbf{(D) } \text{mode} \qquad \textbf{(E) } \text{mid-range}$

Solutions

Solution 1

When they score a $40$ on the next game, the range increases from $73-42=31$ to $73-40=33$. This means the $\boxed{\textbf{(A) } \text{range}}$ increased.

Solution 2

Because $40$ is less than the score of every game they've played so far, the measures of center will never rise. Only measures of spread, such as the $\boxed{\textbf{(A)}~\text{range}}$, may increase.


2015 AMC 8 (ProblemsAnswer KeyResources)
Preceded by
Problem 4
Followed by
Problem 6
1 2 3 4 5 6 7 8 9 10 11 12 13 14 15 16 17 18 19 20 21 22 23 24 25
All AJHSME/AMC 8 Problems and Solutions

The problems on this page are copyrighted by the Mathematical Association of America's American Mathematics Competitions. AMC logo.png